tian.z.zheng
Thanks Received: 0
Vinny Gambini
Vinny Gambini
 
Posts: 11
Joined: October 12th, 2009
 
 
 

Q26 - Political theorist: For all of

by tian.z.zheng Thu Oct 22, 2009 4:20 pm

Using the process of elimination, I go the correct answer choice, which is C.

I diagramed the stimulus and found "TAE --> not EU" to be the missing link. However, I don't see how that corresponds to (c). Even the contrapositive doesn't cut it.

TAE - all agree that it threats their alliance's economy
EU - European Union

Thanks!
 
aileenann
Thanks Received: 227
Atticus Finch
Atticus Finch
 
Posts: 300
Joined: March 10th, 2009
 
This post thanked 2 times.
 
 

Re: Q26 - Political theorist: For all of

by aileenann Sat Oct 24, 2009 11:37 am

Hi and thanks for your question!

This is an assumption question, so we need to figure out what must be true for the argument to work logically. In doing so, we want to figure out which assumption both supports the conclusion and serves a logically necessary role in the argument. Finally, a good way to test out answers is that if you negate an assumption of the argument, it falls apart. Therefore the correct answer, if negated, should ruin the argument.

I am a little confused about your diagramming, but if I understood it correctly it may not be quite the correct translation of the text. You wrote "TAE --> not EU," by which I think you meant "If it threatens all their economies, the EU will not all agree." However, I think that is not what comes from translating the sentence (roughly paraphraed by me) of "Not all EU countries will see a grave threat unless they all agree that it is a threat to the alliance's economy." Knowing that the general statement A unless B translates to ~A ---> B, I can write this in logical terms as:

IF the EU countries do all agree there is a grave threat, THEN they do all agree there is a threat to the alliance's economy.

I hope that helps with the diagramming.

Now let's get to the meat of your question. Remember, an assumption doesn't have to follow from the diagram but rather it has to in some way fill a logical problem with the original argument to make that argument better. Here (C) does just that because it makes the premises more reasonable. It does so by suggesting that there will be situations just as the argument foresees in which the countries will not all see a threat to the economy and hence a grave threat. It makes that premise more relevant by saying that it will actually happen.

I hope that answers your question. Feel free to follow up!
 
tian.z.zheng
Thanks Received: 0
Vinny Gambini
Vinny Gambini
 
Posts: 11
Joined: October 12th, 2009
 
 
 

Re: PT 44, S4, 26, Political theorist

by tian.z.zheng Sun Oct 25, 2009 10:55 pm

Ohh ok.. I see what you mean. Thanks for your detailed explanation.
 
itzakadoozie412
Thanks Received: 0
Vinny Gambini
Vinny Gambini
 
Posts: 17
Joined: September 13th, 2010
 
 
 

Re: PT 44, S4, 26, Political theorist

by itzakadoozie412 Mon Nov 29, 2010 8:05 pm

I see why C is correct, but was initially thrown off b/c I thought the conclusion of the argument was "not all members of the EU will be strong in foreign policy" I guess this point has no place int he argument? I picked D as my first choice b/c I thought this was the conclusion.
User avatar
 
ManhattanPrepLSAT1
Thanks Received: 1909
Atticus Finch
Atticus Finch
 
Posts: 2851
Joined: October 07th, 2009
 
This post thanked 1 time.
 
 

Re: PT 44, S4, 26, Political theorist

by ManhattanPrepLSAT1 Wed Dec 01, 2010 5:45 am

Nope! You have the conclusion right...

If you like formal notation, this is way more excessive then I would ever consider using on the test, but if you can follow this you can really see the logic underneath.

(M-->S) --> RA
RA --> (M-->PPG)
(M-->PPG) --> (M-->TE)
----------------------------
~(M-->S)

(Notation Key: M = members, S = strong, RA = respond aggressively, PPG = perceive problems as grave, TE = threatens economy)

You can simply see it in this format

A --> B
B --> C
C --> D
---------
~A

all you need to do is trigger the contrapositive starting with the last premise. ~D will serve as a sufficient assumption. In this case, ~D is ~(M-->TE), which is exactly what answer choice (C) is saying.

Not sure if that makes things more confusing, let me know!
 
itzakadoozie412
Thanks Received: 0
Vinny Gambini
Vinny Gambini
 
Posts: 17
Joined: September 13th, 2010
 
 
 

Re: PT 44, S4, 26, Political theorist

by itzakadoozie412 Sun Jan 02, 2011 10:57 pm

Im following you logic, but still don't know why D in incorrect. Is it because of 'generally' in the answer choice?
User avatar
 
ManhattanPrepLSAT1
Thanks Received: 1909
Atticus Finch
Atticus Finch
 
Posts: 2851
Joined: October 07th, 2009
 
This post thanked 3 times.
 
 

Re: PT44, S4, Q26 - Political theorist: For all of

by ManhattanPrepLSAT1 Mon Jan 03, 2011 7:34 pm

The biggest issue with answer choice (D) is that it's a quantified relationship. You're right that the word "generally" creates problems. It's not that the word "generally" is too strong or too weak, but rather that we don't want the answer choice to be a conditional relationship when the conclusion is a statement of fact. As is, answer choice (D) won't establish the conclusion because we don't actually know that there are some countries that fail to perceive the economic relevance of problems.

Another big issue, is that it simply skips the evidence in it's attempt to reach the conclusion. it doesn't bridge the gap in the argument, but skips the argument all together and then tries to sound relevant by muddling the claims. "Perceiving the economic relevance" is not the same as "perceiving the problems as grave" or agreeing that the problems "threaten their alliance's economy."

Let me know if you still see it differently though!
 
jamiejames
Thanks Received: 3
Atticus Finch
Atticus Finch
 
Posts: 116
Joined: September 17th, 2011
 
 
 

Re: Q26 - Political theorist: For all of

by jamiejames Thu Mar 29, 2012 6:11 pm

I'll be totally honest here, I didn't do any conditional logic at all with this one, and still got C.

I read the prompt as (without a lot of the extra words and information): An alliance will respond aggressively only if all members perceive the problem as grave. Furthermore, they won't all agree that a threat is grave unless they agree it threatens the alliance's economy.

Therefore, not all members of the alliance will be aggressive.

I prephrased my answer as:

The conclusion is assuming, without warrant, that they won't all agree that a specific threat threatens the alliances economy.

Looked through answer choices, and C jumped out at me. If you negate C: "Problems that appear to some members countries of the EU to threaten the alliance's economy will appear so to others," then the conclusion that they won't all respond aggressively is destroyed.
 
timmydoeslsat
Thanks Received: 887
Atticus Finch
Atticus Finch
 
Posts: 1136
Joined: June 20th, 2011
 
 
trophy
Most Thanked
trophy
First Responder
 

Re: Q26 - Political theorist: For all of

by timmydoeslsat Fri Mar 30, 2012 6:34 pm

jeastman Wrote:I'll be totally honest here, I didn't do any conditional logic at all with this one, and still got C.

I read the prompt as (without a lot of the extra words and information): An alliance will respond aggressively only if all members perceive the problem as grave. Furthermore, they won't all agree that a threat is grave unless they agree it threatens the alliance's economy.

Therefore, not all members of the alliance will be aggressive.

I prephrased my answer as:

The conclusion is assuming, without warrant, that they won't all agree that a specific threat threatens the alliances economy.

Looked through answer choices, and C jumped out at me. If you negate C: "Problems that appear to some members countries of the EU to threaten the alliance's economy will appear so to others," then the conclusion that they won't all respond aggressively is destroyed.


I would advise against using the negation test on a sufficient assumption question stem. We absolutely want to use it on a necessary assumption question, but not on these.

You may be able to go without diagramming this one due to the fact that it clearly follows a long chain of logic, as Matt showed earlier, and then the conclusion states that some members of the EU will be strong on foreign policy.

For this question, you see where the conclusion mentions the EU countries. We are looking for EU countries in the answer choice. Simply using the word countries is not going to definitely prove the conclusion. You could say that "countries" may be referring to those in Africa or North America.

That limits it to C and D.

The reason D is not correct is due to us not knowing if there are EU countries that fail to perceive relevance.

A ---> B ---> C ---> D
____________________
~ A

Choice (C) basically chooses one of those variables to give the negation of to lead us to ~A.
 
magic.imango
Thanks Received: 2
Vinny Gambini
Vinny Gambini
 
Posts: 22
Joined: July 12th, 2014
 
 
 

Re: PT 44, S4, 26, Political theorist

by magic.imango Tue Sep 23, 2014 5:15 pm

mattsherman Wrote:Nope! You have the conclusion right...

If you like formal notation, this is way more excessive then I would ever consider using on the test, but if you can follow this you can really see the logic underneath.

(M-->S) --> RA
RA --> (M-->PPG)
(M-->PPG) --> (M-->TE)
----------------------------
~(M-->S)

(Notation Key: M = members, S = strong, RA = respond aggressively, PPG = perceive problems as grave, TE = threatens economy)

You can simply see it in this format

A --> B
B --> C
C --> D
---------
~A

all you need to do is trigger the contrapositive starting with the last premise. ~D will serve as a sufficient assumption. In this case, ~D is ~(M-->TE), which is exactly what answer choice (C) is saying.

Not sure if that makes things more confusing, let me know!



I read the conclusion as: "not all the members will be strong in foreign policy" so I took that to mean: "M some ~S". Am I incorrect in diagramming it this way?
 
mitrakhanom1
Thanks Received: 1
Elle Woods
Elle Woods
 
Posts: 63
Joined: May 14th, 2013
 
 
 

Re: Q26 - Political theorist: For all of

by mitrakhanom1 Fri May 22, 2015 7:11 pm

I'm confused how Matt Sherman got the last premise:

(M-->PPG) --> (M-->TE)

(Notation Key: M = members, S = strong, RA = respond aggressively, PPG = perceive problems as grave, TE = threatens economy)

My conditional statements were completely different than his and I was unable to draw any conclusions. This feels like extreme manipulation of conditionals. I don't know when your suppose to add parenthesis around the conditionals to manipulate the outcome. Can somebody break it down and explain this to me please?

I got:

alliance-> problem is grave
-alliance--> problem is grave
User avatar
 
maryadkins
Thanks Received: 640
Atticus Finch
Atticus Finch
 
Posts: 1261
Joined: March 23rd, 2011
 
 
 

Re: Q26 - Political theorist: For all of

by maryadkins Sun Jun 14, 2015 6:39 pm

Let me know if this is clearer:

strong --> respond aggressively

respond aggressively --> all perceive problems as grave

all EU members perceive problems as grave --> agree threatens alliance's economy

CONCLUSION:

not all of the EU members will be strong

So how do we get from all of the top part to that conclusion?

Well, first we have to notice that the conclusion is the negation of the first part of the first conditional. We need to trigger a string of conditionals to make this true, for sure. The way to do that is to form a chain that will land us on a negation of that part (i.e. the EU is NOT strong).

So if we have the whole thing backwards, we'd get to it:

DOES NOT agree threatens alliance's economy --> NOT ALL perceive as grave --> WILL NOT respond aggressively --> is NOT strong

(C) triggers it! It triggers that first part.

I'm not sure what you mean by:

mitrakhanom1 Wrote:alliance-> problem is grave
-alliance--> problem is grave


But this is how this problem is done.